How to linearize a constraint including product of two binary variables in summation with different indexes?












0












$begingroup$


I am trying to linearize the following two expressions:



$sum_{k=1}^K sum_{t=1}^Tsum_{h=1}^W x_{ijkt} a_{hjt} =sum_{k=1}^K sum_{t=1}^T x_{ijkt} k , iin N, j in M$



$sum_{k=1}^K sum_{t=p_{ijk}}^Tsum_{l=t-P_{ijk}+1}^t x_{ijkt} a_{hjl} =sum_{k=1}^K sum_{t=p_{ijk}}^T x_{ijkt} a_{hjt} p_{ijk} , iin N, j in M, h in W$



$x_{ijkt}$: binary variable



$a_{hjt}$: binary variable



$p_{ijk}$: parameter



K: parameter



I already know product of two binary variables can be linearized as follows:



ab=z



$a le z$



$b le z$



$zge a+b-1$



Accordingly I did as follows to linearized the first constraint:



$x_{ijkt} a_{hjt}=z_{ijkth}$



$i in N, j in M, k in K, t in T, h in w, x_{ijkt} le z_{ijkth}$



$i in N, j in M, k in K, t in T, h in w, a_{hjt} le z_{ijkth}$



$i in N, j in M, k in K, t in T, h in w, x_{ijkt} + a_{hjt}-1 ge z_{ijkth}$



And finally converted the constraint as follows:



$sum_{k=1}^K sum_{t=1}^Tsum_{h=1}^W z_{ijkth} =sum_{k=1}^K sum_{t=1}^T x_{ijkt} k , iin N, j in M$



However it makes the model infeasible!










share|cite|improve this question









$endgroup$












  • $begingroup$
    Your binary multiplication $z=ab$ is not correct. The constraints $ale z$ and $b le z$ should read $z le a $ and $z le b$.
    $endgroup$
    – Erwin Kalvelagen
    Dec 13 '17 at 2:38


















0












$begingroup$


I am trying to linearize the following two expressions:



$sum_{k=1}^K sum_{t=1}^Tsum_{h=1}^W x_{ijkt} a_{hjt} =sum_{k=1}^K sum_{t=1}^T x_{ijkt} k , iin N, j in M$



$sum_{k=1}^K sum_{t=p_{ijk}}^Tsum_{l=t-P_{ijk}+1}^t x_{ijkt} a_{hjl} =sum_{k=1}^K sum_{t=p_{ijk}}^T x_{ijkt} a_{hjt} p_{ijk} , iin N, j in M, h in W$



$x_{ijkt}$: binary variable



$a_{hjt}$: binary variable



$p_{ijk}$: parameter



K: parameter



I already know product of two binary variables can be linearized as follows:



ab=z



$a le z$



$b le z$



$zge a+b-1$



Accordingly I did as follows to linearized the first constraint:



$x_{ijkt} a_{hjt}=z_{ijkth}$



$i in N, j in M, k in K, t in T, h in w, x_{ijkt} le z_{ijkth}$



$i in N, j in M, k in K, t in T, h in w, a_{hjt} le z_{ijkth}$



$i in N, j in M, k in K, t in T, h in w, x_{ijkt} + a_{hjt}-1 ge z_{ijkth}$



And finally converted the constraint as follows:



$sum_{k=1}^K sum_{t=1}^Tsum_{h=1}^W z_{ijkth} =sum_{k=1}^K sum_{t=1}^T x_{ijkt} k , iin N, j in M$



However it makes the model infeasible!










share|cite|improve this question









$endgroup$












  • $begingroup$
    Your binary multiplication $z=ab$ is not correct. The constraints $ale z$ and $b le z$ should read $z le a $ and $z le b$.
    $endgroup$
    – Erwin Kalvelagen
    Dec 13 '17 at 2:38
















0












0








0





$begingroup$


I am trying to linearize the following two expressions:



$sum_{k=1}^K sum_{t=1}^Tsum_{h=1}^W x_{ijkt} a_{hjt} =sum_{k=1}^K sum_{t=1}^T x_{ijkt} k , iin N, j in M$



$sum_{k=1}^K sum_{t=p_{ijk}}^Tsum_{l=t-P_{ijk}+1}^t x_{ijkt} a_{hjl} =sum_{k=1}^K sum_{t=p_{ijk}}^T x_{ijkt} a_{hjt} p_{ijk} , iin N, j in M, h in W$



$x_{ijkt}$: binary variable



$a_{hjt}$: binary variable



$p_{ijk}$: parameter



K: parameter



I already know product of two binary variables can be linearized as follows:



ab=z



$a le z$



$b le z$



$zge a+b-1$



Accordingly I did as follows to linearized the first constraint:



$x_{ijkt} a_{hjt}=z_{ijkth}$



$i in N, j in M, k in K, t in T, h in w, x_{ijkt} le z_{ijkth}$



$i in N, j in M, k in K, t in T, h in w, a_{hjt} le z_{ijkth}$



$i in N, j in M, k in K, t in T, h in w, x_{ijkt} + a_{hjt}-1 ge z_{ijkth}$



And finally converted the constraint as follows:



$sum_{k=1}^K sum_{t=1}^Tsum_{h=1}^W z_{ijkth} =sum_{k=1}^K sum_{t=1}^T x_{ijkt} k , iin N, j in M$



However it makes the model infeasible!










share|cite|improve this question









$endgroup$




I am trying to linearize the following two expressions:



$sum_{k=1}^K sum_{t=1}^Tsum_{h=1}^W x_{ijkt} a_{hjt} =sum_{k=1}^K sum_{t=1}^T x_{ijkt} k , iin N, j in M$



$sum_{k=1}^K sum_{t=p_{ijk}}^Tsum_{l=t-P_{ijk}+1}^t x_{ijkt} a_{hjl} =sum_{k=1}^K sum_{t=p_{ijk}}^T x_{ijkt} a_{hjt} p_{ijk} , iin N, j in M, h in W$



$x_{ijkt}$: binary variable



$a_{hjt}$: binary variable



$p_{ijk}$: parameter



K: parameter



I already know product of two binary variables can be linearized as follows:



ab=z



$a le z$



$b le z$



$zge a+b-1$



Accordingly I did as follows to linearized the first constraint:



$x_{ijkt} a_{hjt}=z_{ijkth}$



$i in N, j in M, k in K, t in T, h in w, x_{ijkt} le z_{ijkth}$



$i in N, j in M, k in K, t in T, h in w, a_{hjt} le z_{ijkth}$



$i in N, j in M, k in K, t in T, h in w, x_{ijkt} + a_{hjt}-1 ge z_{ijkth}$



And finally converted the constraint as follows:



$sum_{k=1}^K sum_{t=1}^Tsum_{h=1}^W z_{ijkth} =sum_{k=1}^K sum_{t=1}^T x_{ijkt} k , iin N, j in M$



However it makes the model infeasible!







optimization linear-programming linearization






share|cite|improve this question













share|cite|improve this question











share|cite|improve this question




share|cite|improve this question










asked Dec 13 '17 at 2:19









araz nasirianaraz nasirian

11




11












  • $begingroup$
    Your binary multiplication $z=ab$ is not correct. The constraints $ale z$ and $b le z$ should read $z le a $ and $z le b$.
    $endgroup$
    – Erwin Kalvelagen
    Dec 13 '17 at 2:38




















  • $begingroup$
    Your binary multiplication $z=ab$ is not correct. The constraints $ale z$ and $b le z$ should read $z le a $ and $z le b$.
    $endgroup$
    – Erwin Kalvelagen
    Dec 13 '17 at 2:38


















$begingroup$
Your binary multiplication $z=ab$ is not correct. The constraints $ale z$ and $b le z$ should read $z le a $ and $z le b$.
$endgroup$
– Erwin Kalvelagen
Dec 13 '17 at 2:38






$begingroup$
Your binary multiplication $z=ab$ is not correct. The constraints $ale z$ and $b le z$ should read $z le a $ and $z le b$.
$endgroup$
– Erwin Kalvelagen
Dec 13 '17 at 2:38












1 Answer
1






active

oldest

votes


















0












$begingroup$

If you want to handle the sum of products



$$ sum_{i,j} x_i y_j $$



with $x_i, y_i in {0,1}$ you need to introduce a new variable $z_{i,j}in {0,1}$ and write:



$$
begin{align}
&sum_{i,j} z_{i,j}\
&z_{i,j} le x_i&forall i,j\
&z_{i,j} le y_j&forall i,j\
&z_{i,j} ge x_i+y_j-1&forall i,j\
& 0 leq z_{i,j} leq 1&forall i,j
end{align}
$$



Note that we can relax $z$ to be continuous (i.e., $zin [0,1]$) as $z$ assumes integer values automatically. This formulation extends naturally to more indices.






share|cite|improve this answer











$endgroup$













  • $begingroup$
    In the last constraint the direction should be $ge$.
    $endgroup$
    – YukiJ
    Sep 27 '18 at 7:50










  • $begingroup$
    @YukiJ Yes, thanks.
    $endgroup$
    – Erwin Kalvelagen
    Sep 27 '18 at 8:40











Your Answer





StackExchange.ifUsing("editor", function () {
return StackExchange.using("mathjaxEditing", function () {
StackExchange.MarkdownEditor.creationCallbacks.add(function (editor, postfix) {
StackExchange.mathjaxEditing.prepareWmdForMathJax(editor, postfix, [["$", "$"], ["\\(","\\)"]]);
});
});
}, "mathjax-editing");

StackExchange.ready(function() {
var channelOptions = {
tags: "".split(" "),
id: "69"
};
initTagRenderer("".split(" "), "".split(" "), channelOptions);

StackExchange.using("externalEditor", function() {
// Have to fire editor after snippets, if snippets enabled
if (StackExchange.settings.snippets.snippetsEnabled) {
StackExchange.using("snippets", function() {
createEditor();
});
}
else {
createEditor();
}
});

function createEditor() {
StackExchange.prepareEditor({
heartbeatType: 'answer',
autoActivateHeartbeat: false,
convertImagesToLinks: true,
noModals: true,
showLowRepImageUploadWarning: true,
reputationToPostImages: 10,
bindNavPrevention: true,
postfix: "",
imageUploader: {
brandingHtml: "Powered by u003ca class="icon-imgur-white" href="https://imgur.com/"u003eu003c/au003e",
contentPolicyHtml: "User contributions licensed under u003ca href="https://creativecommons.org/licenses/by-sa/3.0/"u003ecc by-sa 3.0 with attribution requiredu003c/au003e u003ca href="https://stackoverflow.com/legal/content-policy"u003e(content policy)u003c/au003e",
allowUrls: true
},
noCode: true, onDemand: true,
discardSelector: ".discard-answer"
,immediatelyShowMarkdownHelp:true
});


}
});














draft saved

draft discarded


















StackExchange.ready(
function () {
StackExchange.openid.initPostLogin('.new-post-login', 'https%3a%2f%2fmath.stackexchange.com%2fquestions%2f2564249%2fhow-to-linearize-a-constraint-including-product-of-two-binary-variables-in-summa%23new-answer', 'question_page');
}
);

Post as a guest















Required, but never shown

























1 Answer
1






active

oldest

votes








1 Answer
1






active

oldest

votes









active

oldest

votes






active

oldest

votes









0












$begingroup$

If you want to handle the sum of products



$$ sum_{i,j} x_i y_j $$



with $x_i, y_i in {0,1}$ you need to introduce a new variable $z_{i,j}in {0,1}$ and write:



$$
begin{align}
&sum_{i,j} z_{i,j}\
&z_{i,j} le x_i&forall i,j\
&z_{i,j} le y_j&forall i,j\
&z_{i,j} ge x_i+y_j-1&forall i,j\
& 0 leq z_{i,j} leq 1&forall i,j
end{align}
$$



Note that we can relax $z$ to be continuous (i.e., $zin [0,1]$) as $z$ assumes integer values automatically. This formulation extends naturally to more indices.






share|cite|improve this answer











$endgroup$













  • $begingroup$
    In the last constraint the direction should be $ge$.
    $endgroup$
    – YukiJ
    Sep 27 '18 at 7:50










  • $begingroup$
    @YukiJ Yes, thanks.
    $endgroup$
    – Erwin Kalvelagen
    Sep 27 '18 at 8:40
















0












$begingroup$

If you want to handle the sum of products



$$ sum_{i,j} x_i y_j $$



with $x_i, y_i in {0,1}$ you need to introduce a new variable $z_{i,j}in {0,1}$ and write:



$$
begin{align}
&sum_{i,j} z_{i,j}\
&z_{i,j} le x_i&forall i,j\
&z_{i,j} le y_j&forall i,j\
&z_{i,j} ge x_i+y_j-1&forall i,j\
& 0 leq z_{i,j} leq 1&forall i,j
end{align}
$$



Note that we can relax $z$ to be continuous (i.e., $zin [0,1]$) as $z$ assumes integer values automatically. This formulation extends naturally to more indices.






share|cite|improve this answer











$endgroup$













  • $begingroup$
    In the last constraint the direction should be $ge$.
    $endgroup$
    – YukiJ
    Sep 27 '18 at 7:50










  • $begingroup$
    @YukiJ Yes, thanks.
    $endgroup$
    – Erwin Kalvelagen
    Sep 27 '18 at 8:40














0












0








0





$begingroup$

If you want to handle the sum of products



$$ sum_{i,j} x_i y_j $$



with $x_i, y_i in {0,1}$ you need to introduce a new variable $z_{i,j}in {0,1}$ and write:



$$
begin{align}
&sum_{i,j} z_{i,j}\
&z_{i,j} le x_i&forall i,j\
&z_{i,j} le y_j&forall i,j\
&z_{i,j} ge x_i+y_j-1&forall i,j\
& 0 leq z_{i,j} leq 1&forall i,j
end{align}
$$



Note that we can relax $z$ to be continuous (i.e., $zin [0,1]$) as $z$ assumes integer values automatically. This formulation extends naturally to more indices.






share|cite|improve this answer











$endgroup$



If you want to handle the sum of products



$$ sum_{i,j} x_i y_j $$



with $x_i, y_i in {0,1}$ you need to introduce a new variable $z_{i,j}in {0,1}$ and write:



$$
begin{align}
&sum_{i,j} z_{i,j}\
&z_{i,j} le x_i&forall i,j\
&z_{i,j} le y_j&forall i,j\
&z_{i,j} ge x_i+y_j-1&forall i,j\
& 0 leq z_{i,j} leq 1&forall i,j
end{align}
$$



Note that we can relax $z$ to be continuous (i.e., $zin [0,1]$) as $z$ assumes integer values automatically. This formulation extends naturally to more indices.







share|cite|improve this answer














share|cite|improve this answer



share|cite|improve this answer








edited Sep 27 '18 at 8:38









YukiJ

2,1112928




2,1112928










answered Dec 13 '17 at 10:21









Erwin KalvelagenErwin Kalvelagen

3,0892511




3,0892511












  • $begingroup$
    In the last constraint the direction should be $ge$.
    $endgroup$
    – YukiJ
    Sep 27 '18 at 7:50










  • $begingroup$
    @YukiJ Yes, thanks.
    $endgroup$
    – Erwin Kalvelagen
    Sep 27 '18 at 8:40


















  • $begingroup$
    In the last constraint the direction should be $ge$.
    $endgroup$
    – YukiJ
    Sep 27 '18 at 7:50










  • $begingroup$
    @YukiJ Yes, thanks.
    $endgroup$
    – Erwin Kalvelagen
    Sep 27 '18 at 8:40
















$begingroup$
In the last constraint the direction should be $ge$.
$endgroup$
– YukiJ
Sep 27 '18 at 7:50




$begingroup$
In the last constraint the direction should be $ge$.
$endgroup$
– YukiJ
Sep 27 '18 at 7:50












$begingroup$
@YukiJ Yes, thanks.
$endgroup$
– Erwin Kalvelagen
Sep 27 '18 at 8:40




$begingroup$
@YukiJ Yes, thanks.
$endgroup$
– Erwin Kalvelagen
Sep 27 '18 at 8:40


















draft saved

draft discarded




















































Thanks for contributing an answer to Mathematics Stack Exchange!


  • Please be sure to answer the question. Provide details and share your research!

But avoid



  • Asking for help, clarification, or responding to other answers.

  • Making statements based on opinion; back them up with references or personal experience.


Use MathJax to format equations. MathJax reference.


To learn more, see our tips on writing great answers.




draft saved


draft discarded














StackExchange.ready(
function () {
StackExchange.openid.initPostLogin('.new-post-login', 'https%3a%2f%2fmath.stackexchange.com%2fquestions%2f2564249%2fhow-to-linearize-a-constraint-including-product-of-two-binary-variables-in-summa%23new-answer', 'question_page');
}
);

Post as a guest















Required, but never shown





















































Required, but never shown














Required, but never shown












Required, but never shown







Required, but never shown

































Required, but never shown














Required, but never shown












Required, but never shown







Required, but never shown







Popular posts from this blog

Can a sorcerer learn a 5th-level spell early by creating spell slots using the Font of Magic feature?

Does disintegrating a polymorphed enemy still kill it after the 2018 errata?

A Topological Invariant for $pi_3(U(n))$